Finding Vectors for Cross Product Problem - Explanation and Solution

Click For Summary
To find vectors v such that <1,2,1> X v = <3,1,-5>, the equations derived from the cross product lead to two independent equations with three variables, indicating one variable can be arbitrary. The equations simplify to a consistent solution, confirming the existence of such vectors. In contrast, for <1,2,1> X v = <3,1,5>, the resulting equations are inconsistent, showing no solution exists. The discussion emphasizes understanding the independence of equations in vector problems. The clarification on the nature of solutions is crucial for solving cross product problems effectively.
JNBirDy
Messages
37
Reaction score
0

Homework Statement


i) Find all vectors v such that <1,2,1> X v = <3,1,-5>

ii) Explain why there is no vector v such that <1,2,1> X v = <3,1,5>

Homework Equations



a X b = <a_{2}b_{3} - a_{3}b_{2}, a_{3}b_{1} - a_{1}b_{3}, a_{1}b_{2} - a_{2}b_{1})

The Attempt at a Solution



i)
<1,2,1> X v
= <2v_{3} -v_{2}, v_{1} - v_{3}, v_{2} - 2v_{1}>

Which leaves me with three equations:
2v_{3} -v_{2} = 3
v_{1} - v_{3} = 1
v_{2} - 2v_{1} = -5

Now, every time I try to fiddle with them, I end up with 0 = 0.

Any help?
 
Physics news on Phys.org
Express v2 from the first equation and v1 from the second one. v3 cancels when you substitute for v1 and v2 in the third equation, and you get -5=-5. This means that the three equations are not independent, and you have two equations for three variables: one variable is arbitrary.

ehild
 
ehild said:
Express v2 from the first equation and v1 from the second one. v3 cancels when you substitute for v1 and v2 in the third equation, and you get -5=-5. This means that the three equations are not independent, and you have two equations for three variables: one variable is arbitrary.

ehild

Ah, yes, I think I get it now. For some reason I was thinking that I needed to get actual numbers.

Thanks.
 
Question: A clock's minute hand has length 4 and its hour hand has length 3. What is the distance between the tips at the moment when it is increasing most rapidly?(Putnam Exam Question) Answer: Making assumption that both the hands moves at constant angular velocities, the answer is ## \sqrt{7} .## But don't you think this assumption is somewhat doubtful and wrong?

Similar threads

  • · Replies 14 ·
Replies
14
Views
2K
  • · Replies 2 ·
Replies
2
Views
2K
  • · Replies 3 ·
Replies
3
Views
1K
  • · Replies 9 ·
Replies
9
Views
2K
  • · Replies 7 ·
Replies
7
Views
2K
  • · Replies 11 ·
Replies
11
Views
2K
  • · Replies 2 ·
Replies
2
Views
2K
  • · Replies 5 ·
Replies
5
Views
1K
Replies
6
Views
1K
  • · Replies 8 ·
Replies
8
Views
2K